What is the cardinality of the following set?












0












$begingroup$


What is the cardinality of the following set: ${f: mathbb{R} rightarrow {a+bsqrt[3]4 | a,b in mathbb{Q}}}$?



Let $S = {f: mathbb{R} rightarrow {a+bsqrt[3]4 | a,b in mathbb{Q}}}$.



I already show that ${f:mathbb{R}rightarrow{0,1}} subset S$, then $|S| ge |{f:mathbb{R}rightarrow{0,1}}| = 2^c$, and I need to show the rest of the part i.e.: $|S| le 2^c$, then to conclude that the cardinality of the set $S$ is $2^c$. (please correct me if this part is wrong)



But I get stuck, I don't know how to show that $|S| le 2^c$. Can you please help me out? Thanks in advance!










share|cite|improve this question











$endgroup$












  • $begingroup$
    I assume $R$ and $Q$ are supposed to be $mathbb{R}$ and $mathbb{Q}$?
    $endgroup$
    – GenericMathematician
    Dec 9 '18 at 21:57






  • 2




    $begingroup$
    Would you be more comfortable if ${,a+bsqrt[3]4mid a,binBbb Q,}$ were replaced with $Bbb N$?
    $endgroup$
    – Hagen von Eitzen
    Dec 9 '18 at 21:57


















0












$begingroup$


What is the cardinality of the following set: ${f: mathbb{R} rightarrow {a+bsqrt[3]4 | a,b in mathbb{Q}}}$?



Let $S = {f: mathbb{R} rightarrow {a+bsqrt[3]4 | a,b in mathbb{Q}}}$.



I already show that ${f:mathbb{R}rightarrow{0,1}} subset S$, then $|S| ge |{f:mathbb{R}rightarrow{0,1}}| = 2^c$, and I need to show the rest of the part i.e.: $|S| le 2^c$, then to conclude that the cardinality of the set $S$ is $2^c$. (please correct me if this part is wrong)



But I get stuck, I don't know how to show that $|S| le 2^c$. Can you please help me out? Thanks in advance!










share|cite|improve this question











$endgroup$












  • $begingroup$
    I assume $R$ and $Q$ are supposed to be $mathbb{R}$ and $mathbb{Q}$?
    $endgroup$
    – GenericMathematician
    Dec 9 '18 at 21:57






  • 2




    $begingroup$
    Would you be more comfortable if ${,a+bsqrt[3]4mid a,binBbb Q,}$ were replaced with $Bbb N$?
    $endgroup$
    – Hagen von Eitzen
    Dec 9 '18 at 21:57
















0












0








0


1



$begingroup$


What is the cardinality of the following set: ${f: mathbb{R} rightarrow {a+bsqrt[3]4 | a,b in mathbb{Q}}}$?



Let $S = {f: mathbb{R} rightarrow {a+bsqrt[3]4 | a,b in mathbb{Q}}}$.



I already show that ${f:mathbb{R}rightarrow{0,1}} subset S$, then $|S| ge |{f:mathbb{R}rightarrow{0,1}}| = 2^c$, and I need to show the rest of the part i.e.: $|S| le 2^c$, then to conclude that the cardinality of the set $S$ is $2^c$. (please correct me if this part is wrong)



But I get stuck, I don't know how to show that $|S| le 2^c$. Can you please help me out? Thanks in advance!










share|cite|improve this question











$endgroup$




What is the cardinality of the following set: ${f: mathbb{R} rightarrow {a+bsqrt[3]4 | a,b in mathbb{Q}}}$?



Let $S = {f: mathbb{R} rightarrow {a+bsqrt[3]4 | a,b in mathbb{Q}}}$.



I already show that ${f:mathbb{R}rightarrow{0,1}} subset S$, then $|S| ge |{f:mathbb{R}rightarrow{0,1}}| = 2^c$, and I need to show the rest of the part i.e.: $|S| le 2^c$, then to conclude that the cardinality of the set $S$ is $2^c$. (please correct me if this part is wrong)



But I get stuck, I don't know how to show that $|S| le 2^c$. Can you please help me out? Thanks in advance!







abstract-algebra elementary-set-theory cardinals






share|cite|improve this question















share|cite|improve this question













share|cite|improve this question




share|cite|improve this question








edited Dec 9 '18 at 22:08









ODF

1,486510




1,486510










asked Dec 9 '18 at 21:50









EdwardEdward

132




132












  • $begingroup$
    I assume $R$ and $Q$ are supposed to be $mathbb{R}$ and $mathbb{Q}$?
    $endgroup$
    – GenericMathematician
    Dec 9 '18 at 21:57






  • 2




    $begingroup$
    Would you be more comfortable if ${,a+bsqrt[3]4mid a,binBbb Q,}$ were replaced with $Bbb N$?
    $endgroup$
    – Hagen von Eitzen
    Dec 9 '18 at 21:57




















  • $begingroup$
    I assume $R$ and $Q$ are supposed to be $mathbb{R}$ and $mathbb{Q}$?
    $endgroup$
    – GenericMathematician
    Dec 9 '18 at 21:57






  • 2




    $begingroup$
    Would you be more comfortable if ${,a+bsqrt[3]4mid a,binBbb Q,}$ were replaced with $Bbb N$?
    $endgroup$
    – Hagen von Eitzen
    Dec 9 '18 at 21:57


















$begingroup$
I assume $R$ and $Q$ are supposed to be $mathbb{R}$ and $mathbb{Q}$?
$endgroup$
– GenericMathematician
Dec 9 '18 at 21:57




$begingroup$
I assume $R$ and $Q$ are supposed to be $mathbb{R}$ and $mathbb{Q}$?
$endgroup$
– GenericMathematician
Dec 9 '18 at 21:57




2




2




$begingroup$
Would you be more comfortable if ${,a+bsqrt[3]4mid a,binBbb Q,}$ were replaced with $Bbb N$?
$endgroup$
– Hagen von Eitzen
Dec 9 '18 at 21:57






$begingroup$
Would you be more comfortable if ${,a+bsqrt[3]4mid a,binBbb Q,}$ were replaced with $Bbb N$?
$endgroup$
– Hagen von Eitzen
Dec 9 '18 at 21:57












1 Answer
1






active

oldest

votes


















0












$begingroup$

You're looking at the set of functins from $mathbb{R}$ which has size $2^{aleph_0}= mathfrak{c}$ to a countable set (as $|mathbb{Q}^2| = aleph_0$) which by definition has size $(aleph_0)^{2^{aleph_0}} = 2^{mathfrak{c}}$ by standard cardinal arithmetic : $$ 2^mathfrak{c} le aleph_0^{mathfrak{c}} le (2^mathfrak{c})^mathfrak{c} = 2^mathfrak{c}$$






share|cite|improve this answer









$endgroup$













  • $begingroup$
    Can you find an one to one function map to show that the cardinality of the set S is less than or equal to $2^c$. Since we haven't learned cardinal arithmetic yet...
    $endgroup$
    – Edward
    Dec 9 '18 at 22:11












  • $begingroup$
    You'll need axiom of choice for that, in order to use a bijection between $mathfrak c times mathfrak c$ and $mathfrak c$.
    $endgroup$
    – mathcounterexamples.net
    Dec 10 '18 at 16:52












  • $begingroup$
    @mathcounterexamples.net well for $mathfrak{c}$ we can show it without AC: use the representation as sequences of natural numbers that can be interleaved.
    $endgroup$
    – Henno Brandsma
    Dec 10 '18 at 17:38











Your Answer





StackExchange.ifUsing("editor", function () {
return StackExchange.using("mathjaxEditing", function () {
StackExchange.MarkdownEditor.creationCallbacks.add(function (editor, postfix) {
StackExchange.mathjaxEditing.prepareWmdForMathJax(editor, postfix, [["$", "$"], ["\\(","\\)"]]);
});
});
}, "mathjax-editing");

StackExchange.ready(function() {
var channelOptions = {
tags: "".split(" "),
id: "69"
};
initTagRenderer("".split(" "), "".split(" "), channelOptions);

StackExchange.using("externalEditor", function() {
// Have to fire editor after snippets, if snippets enabled
if (StackExchange.settings.snippets.snippetsEnabled) {
StackExchange.using("snippets", function() {
createEditor();
});
}
else {
createEditor();
}
});

function createEditor() {
StackExchange.prepareEditor({
heartbeatType: 'answer',
autoActivateHeartbeat: false,
convertImagesToLinks: true,
noModals: true,
showLowRepImageUploadWarning: true,
reputationToPostImages: 10,
bindNavPrevention: true,
postfix: "",
imageUploader: {
brandingHtml: "Powered by u003ca class="icon-imgur-white" href="https://imgur.com/"u003eu003c/au003e",
contentPolicyHtml: "User contributions licensed under u003ca href="https://creativecommons.org/licenses/by-sa/3.0/"u003ecc by-sa 3.0 with attribution requiredu003c/au003e u003ca href="https://stackoverflow.com/legal/content-policy"u003e(content policy)u003c/au003e",
allowUrls: true
},
noCode: true, onDemand: true,
discardSelector: ".discard-answer"
,immediatelyShowMarkdownHelp:true
});


}
});














draft saved

draft discarded


















StackExchange.ready(
function () {
StackExchange.openid.initPostLogin('.new-post-login', 'https%3a%2f%2fmath.stackexchange.com%2fquestions%2f3033071%2fwhat-is-the-cardinality-of-the-following-set%23new-answer', 'question_page');
}
);

Post as a guest















Required, but never shown

























1 Answer
1






active

oldest

votes








1 Answer
1






active

oldest

votes









active

oldest

votes






active

oldest

votes









0












$begingroup$

You're looking at the set of functins from $mathbb{R}$ which has size $2^{aleph_0}= mathfrak{c}$ to a countable set (as $|mathbb{Q}^2| = aleph_0$) which by definition has size $(aleph_0)^{2^{aleph_0}} = 2^{mathfrak{c}}$ by standard cardinal arithmetic : $$ 2^mathfrak{c} le aleph_0^{mathfrak{c}} le (2^mathfrak{c})^mathfrak{c} = 2^mathfrak{c}$$






share|cite|improve this answer









$endgroup$













  • $begingroup$
    Can you find an one to one function map to show that the cardinality of the set S is less than or equal to $2^c$. Since we haven't learned cardinal arithmetic yet...
    $endgroup$
    – Edward
    Dec 9 '18 at 22:11












  • $begingroup$
    You'll need axiom of choice for that, in order to use a bijection between $mathfrak c times mathfrak c$ and $mathfrak c$.
    $endgroup$
    – mathcounterexamples.net
    Dec 10 '18 at 16:52












  • $begingroup$
    @mathcounterexamples.net well for $mathfrak{c}$ we can show it without AC: use the representation as sequences of natural numbers that can be interleaved.
    $endgroup$
    – Henno Brandsma
    Dec 10 '18 at 17:38
















0












$begingroup$

You're looking at the set of functins from $mathbb{R}$ which has size $2^{aleph_0}= mathfrak{c}$ to a countable set (as $|mathbb{Q}^2| = aleph_0$) which by definition has size $(aleph_0)^{2^{aleph_0}} = 2^{mathfrak{c}}$ by standard cardinal arithmetic : $$ 2^mathfrak{c} le aleph_0^{mathfrak{c}} le (2^mathfrak{c})^mathfrak{c} = 2^mathfrak{c}$$






share|cite|improve this answer









$endgroup$













  • $begingroup$
    Can you find an one to one function map to show that the cardinality of the set S is less than or equal to $2^c$. Since we haven't learned cardinal arithmetic yet...
    $endgroup$
    – Edward
    Dec 9 '18 at 22:11












  • $begingroup$
    You'll need axiom of choice for that, in order to use a bijection between $mathfrak c times mathfrak c$ and $mathfrak c$.
    $endgroup$
    – mathcounterexamples.net
    Dec 10 '18 at 16:52












  • $begingroup$
    @mathcounterexamples.net well for $mathfrak{c}$ we can show it without AC: use the representation as sequences of natural numbers that can be interleaved.
    $endgroup$
    – Henno Brandsma
    Dec 10 '18 at 17:38














0












0








0





$begingroup$

You're looking at the set of functins from $mathbb{R}$ which has size $2^{aleph_0}= mathfrak{c}$ to a countable set (as $|mathbb{Q}^2| = aleph_0$) which by definition has size $(aleph_0)^{2^{aleph_0}} = 2^{mathfrak{c}}$ by standard cardinal arithmetic : $$ 2^mathfrak{c} le aleph_0^{mathfrak{c}} le (2^mathfrak{c})^mathfrak{c} = 2^mathfrak{c}$$






share|cite|improve this answer









$endgroup$



You're looking at the set of functins from $mathbb{R}$ which has size $2^{aleph_0}= mathfrak{c}$ to a countable set (as $|mathbb{Q}^2| = aleph_0$) which by definition has size $(aleph_0)^{2^{aleph_0}} = 2^{mathfrak{c}}$ by standard cardinal arithmetic : $$ 2^mathfrak{c} le aleph_0^{mathfrak{c}} le (2^mathfrak{c})^mathfrak{c} = 2^mathfrak{c}$$







share|cite|improve this answer












share|cite|improve this answer



share|cite|improve this answer










answered Dec 9 '18 at 22:03









Henno BrandsmaHenno Brandsma

110k347116




110k347116












  • $begingroup$
    Can you find an one to one function map to show that the cardinality of the set S is less than or equal to $2^c$. Since we haven't learned cardinal arithmetic yet...
    $endgroup$
    – Edward
    Dec 9 '18 at 22:11












  • $begingroup$
    You'll need axiom of choice for that, in order to use a bijection between $mathfrak c times mathfrak c$ and $mathfrak c$.
    $endgroup$
    – mathcounterexamples.net
    Dec 10 '18 at 16:52












  • $begingroup$
    @mathcounterexamples.net well for $mathfrak{c}$ we can show it without AC: use the representation as sequences of natural numbers that can be interleaved.
    $endgroup$
    – Henno Brandsma
    Dec 10 '18 at 17:38


















  • $begingroup$
    Can you find an one to one function map to show that the cardinality of the set S is less than or equal to $2^c$. Since we haven't learned cardinal arithmetic yet...
    $endgroup$
    – Edward
    Dec 9 '18 at 22:11












  • $begingroup$
    You'll need axiom of choice for that, in order to use a bijection between $mathfrak c times mathfrak c$ and $mathfrak c$.
    $endgroup$
    – mathcounterexamples.net
    Dec 10 '18 at 16:52












  • $begingroup$
    @mathcounterexamples.net well for $mathfrak{c}$ we can show it without AC: use the representation as sequences of natural numbers that can be interleaved.
    $endgroup$
    – Henno Brandsma
    Dec 10 '18 at 17:38
















$begingroup$
Can you find an one to one function map to show that the cardinality of the set S is less than or equal to $2^c$. Since we haven't learned cardinal arithmetic yet...
$endgroup$
– Edward
Dec 9 '18 at 22:11






$begingroup$
Can you find an one to one function map to show that the cardinality of the set S is less than or equal to $2^c$. Since we haven't learned cardinal arithmetic yet...
$endgroup$
– Edward
Dec 9 '18 at 22:11














$begingroup$
You'll need axiom of choice for that, in order to use a bijection between $mathfrak c times mathfrak c$ and $mathfrak c$.
$endgroup$
– mathcounterexamples.net
Dec 10 '18 at 16:52






$begingroup$
You'll need axiom of choice for that, in order to use a bijection between $mathfrak c times mathfrak c$ and $mathfrak c$.
$endgroup$
– mathcounterexamples.net
Dec 10 '18 at 16:52














$begingroup$
@mathcounterexamples.net well for $mathfrak{c}$ we can show it without AC: use the representation as sequences of natural numbers that can be interleaved.
$endgroup$
– Henno Brandsma
Dec 10 '18 at 17:38




$begingroup$
@mathcounterexamples.net well for $mathfrak{c}$ we can show it without AC: use the representation as sequences of natural numbers that can be interleaved.
$endgroup$
– Henno Brandsma
Dec 10 '18 at 17:38


















draft saved

draft discarded




















































Thanks for contributing an answer to Mathematics Stack Exchange!


  • Please be sure to answer the question. Provide details and share your research!

But avoid



  • Asking for help, clarification, or responding to other answers.

  • Making statements based on opinion; back them up with references or personal experience.


Use MathJax to format equations. MathJax reference.


To learn more, see our tips on writing great answers.




draft saved


draft discarded














StackExchange.ready(
function () {
StackExchange.openid.initPostLogin('.new-post-login', 'https%3a%2f%2fmath.stackexchange.com%2fquestions%2f3033071%2fwhat-is-the-cardinality-of-the-following-set%23new-answer', 'question_page');
}
);

Post as a guest















Required, but never shown





















































Required, but never shown














Required, but never shown












Required, but never shown







Required, but never shown

































Required, but never shown














Required, but never shown












Required, but never shown







Required, but never shown







Popular posts from this blog

Plaza Victoria

In PowerPoint, is there a keyboard shortcut for bulleted / numbered list?

How to put 3 figures in Latex with 2 figures side by side and 1 below these side by side images but in...